handdator

Visa fullständig version : Fysikproblem


Zakath
2007-04-11, 23:52
Behöver lite hjälp med hur man löser en viss typ av problem som berör kraftmoment.

Ett exempel jag har är detta problem:

En cirkulär sågklinga roterar 42 varv/s när motorn stängs av, på 18s minskas varvtalet till 13 varv/s. Anta att sågklingan kan anses vara en homogen cirkulär skiva med radien 0,13 m och massan 0,42 kg. Beräkna det bromasande kraftmoment som sågklingan påverkas av.

Tröghetsmomentet hos en homogen cirkulär skiva med massan m och radien r är (mr^2)/2

Svaret är: M = (pi*m*r^2)(42-13)/18s = 0,036 Nm


Jag kan verkligen inte komma fram till detta svaret. Stort tack på förhand för alla tips (eller hela lösningen om ni orkar). :)

Nithian
2007-04-12, 00:18
Blir smått deprimerad när jag läser sånt här

Raggis
2007-04-12, 00:19
Ska försöka mig på uppgiften alldeles strax.

Zakath
2007-04-12, 00:21
Ska försöka mig på uppgiften alldeles strax.

*kissass* :)

Dumleman
2007-04-12, 00:28
Deltarörelsemängden men nu gissar jag :).

Zakath
2007-04-12, 00:34
Deltarörelsemängden men nu gissar jag :).

Att jag inte tänkte på det, när jag kolalr lite på formlerna så verkar det faktiskt som att det åtminstone kanske kan bli något. :) Får se vad Raggis säger.

Raggis
2007-04-12, 01:10
Kolla nyss upp deltarörelsemängden, och det ser lovande ut =) Ska testa o räkna lite på det nu (käka + utöva lite kardio innan).

daggermoon
2007-04-12, 07:45
Räkna på rörelsemängdsmomentet ser ut att vara vad man ska göra. Rörelemängdsmomentet är väl lika med tröghetsmomentet multiplicerat med vinkelhastigheten om jag inte är helt snedseglad. Man räknar ut skillnaden i rörelsemängdsmoment och delar med tiden för att få kraftmomentet.
Det var förvisso ett antal år sedan jag läste mekanik så jag skulle ju kunna ha fel :) .

balder
2007-04-12, 09:11
M = [tröghetsmomentet]*[vinkelaccelerationen]

Tröghetsmomentet för en roterande platta var redan given.
Vinkelaccelerationen kan du bestämma som skillnaden i vinkelhastighet genom tiden. Alltså vinkelacc = [(42*2*pi)-(13*2*pi)]/18

Nu har du allt du behöver för att få fram det rätta svaret. (Bryt ut 2*pi)

Zakath
2007-04-12, 10:28
Tack så hemskt mycket för förklaringarna! Nu var det ju jättelätt att lösa uppgiften, nu måste jag bara lära mig det svåra att härleda formeln då den inte finns med i min formelsamling.